A sequence of holomorphic functions in D(0,1)

The name of the pictureThe name of the pictureThe name of the pictureClash Royale CLAN TAG#URR8PPP











up vote
0
down vote

favorite












Let $f_n$ be a sequence of holomorphic functions in the open unit disk $D(0,1)$ such that $Re f_n geq 0$.



How can I show that either $|f_n(z)| rightarrow infty$ as $ n rightarrow infty$ for all $z in D(0,1)$ or $f_n$ has a subsequence which converges uniformly on every compact subset of $D(0,1)$?







share|cite|improve this question





















  • Hint: en.wikipedia.org/wiki/Harnack%27s_inequality
    – David C. Ullrich
    Jul 16 at 16:33














up vote
0
down vote

favorite












Let $f_n$ be a sequence of holomorphic functions in the open unit disk $D(0,1)$ such that $Re f_n geq 0$.



How can I show that either $|f_n(z)| rightarrow infty$ as $ n rightarrow infty$ for all $z in D(0,1)$ or $f_n$ has a subsequence which converges uniformly on every compact subset of $D(0,1)$?







share|cite|improve this question





















  • Hint: en.wikipedia.org/wiki/Harnack%27s_inequality
    – David C. Ullrich
    Jul 16 at 16:33












up vote
0
down vote

favorite









up vote
0
down vote

favorite











Let $f_n$ be a sequence of holomorphic functions in the open unit disk $D(0,1)$ such that $Re f_n geq 0$.



How can I show that either $|f_n(z)| rightarrow infty$ as $ n rightarrow infty$ for all $z in D(0,1)$ or $f_n$ has a subsequence which converges uniformly on every compact subset of $D(0,1)$?







share|cite|improve this question













Let $f_n$ be a sequence of holomorphic functions in the open unit disk $D(0,1)$ such that $Re f_n geq 0$.



How can I show that either $|f_n(z)| rightarrow infty$ as $ n rightarrow infty$ for all $z in D(0,1)$ or $f_n$ has a subsequence which converges uniformly on every compact subset of $D(0,1)$?









share|cite|improve this question












share|cite|improve this question




share|cite|improve this question








edited Jul 26 at 21:19
























asked Jul 16 at 16:11









Rachel.L

375




375











  • Hint: en.wikipedia.org/wiki/Harnack%27s_inequality
    – David C. Ullrich
    Jul 16 at 16:33
















  • Hint: en.wikipedia.org/wiki/Harnack%27s_inequality
    – David C. Ullrich
    Jul 16 at 16:33















Hint: en.wikipedia.org/wiki/Harnack%27s_inequality
– David C. Ullrich
Jul 16 at 16:33




Hint: en.wikipedia.org/wiki/Harnack%27s_inequality
– David C. Ullrich
Jul 16 at 16:33










1 Answer
1






active

oldest

votes

















up vote
0
down vote



accepted










It follows easily from the version of Harnak's inequality here that if $K$ is a compact subset of $Bbb D=D(0,1)$ there exist $c_K$ and $C_K$ such that
$$c_Ku(0)le u(z)le C_Ku(0)quad(zin K)$$for every positive harmonic function $u$ in $Bbb D$.



So if $Re f_n(0)toinfty$ then $|f_n(z)|toinfty$ uniformly on compact sets. Otoh if $Re f_n(0)nottoinfty$ then restricting to a subsequence we can assume that $Re f_n(0)$ is bounded; now the inequality above shows that $e^f_n$ is uniformly bounded on compact sets...



Heh, a slick way to get around unpleasantness with the logarithm for the punchline: So again passing to a subequence we can assume that $e^f_nto g$ uniformly on compact sets. Since $left|e^f_nright|ge1$ it follows that $g$ has no zero, so $g=e^f$. How to get from there to $f_nto f$: Since normal convergence implies normal convergence of the derivatives we have $f_n'e^f_nto f'e^f$, and now since $left|e^f_nright|ge1$ it follows that $f_n'to f'$ uniformly on compact sets. So if we choose our logarithms so that $f_n(0)to f(0)$ it follows that $f_nto f$ uniformly on compact sets.






share|cite|improve this answer























  • Why not just go directly to $e^-f_n $ is uniformly bounded?
    – zhw.
    Jul 16 at 23:46










  • @zhw. This is what they call a dense argument...
    – David C. Ullrich
    Jul 16 at 23:48










  • @zhw. Actually I have a better reason than I did when I wrote the first version. Saw a cute way to handle the technicalities at the end; for that to work with the $e^-f_n$ version I need to know that $|e^-f_n|ge c>0$ on compact sets.
    – David C. Ullrich
    Jul 16 at 23:58











  • @Rachel.L Seriously? $g$ is holomorphic in a disk and has no zero, so it has a holomorphic logarithm...
    – David C. Ullrich
    Jul 24 at 20:57










Your Answer




StackExchange.ifUsing("editor", function ()
return StackExchange.using("mathjaxEditing", function ()
StackExchange.MarkdownEditor.creationCallbacks.add(function (editor, postfix)
StackExchange.mathjaxEditing.prepareWmdForMathJax(editor, postfix, [["$", "$"], ["\\(","\\)"]]);
);
);
, "mathjax-editing");

StackExchange.ready(function()
var channelOptions =
tags: "".split(" "),
id: "69"
;
initTagRenderer("".split(" "), "".split(" "), channelOptions);

StackExchange.using("externalEditor", function()
// Have to fire editor after snippets, if snippets enabled
if (StackExchange.settings.snippets.snippetsEnabled)
StackExchange.using("snippets", function()
createEditor();
);

else
createEditor();

);

function createEditor()
StackExchange.prepareEditor(
heartbeatType: 'answer',
convertImagesToLinks: true,
noModals: false,
showLowRepImageUploadWarning: true,
reputationToPostImages: 10,
bindNavPrevention: true,
postfix: "",
noCode: true, onDemand: true,
discardSelector: ".discard-answer"
,immediatelyShowMarkdownHelp:true
);



);








 

draft saved


draft discarded


















StackExchange.ready(
function ()
StackExchange.openid.initPostLogin('.new-post-login', 'https%3a%2f%2fmath.stackexchange.com%2fquestions%2f2853554%2fa-sequence-of-holomorphic-functions-in-d0-1%23new-answer', 'question_page');

);

Post as a guest






























1 Answer
1






active

oldest

votes








1 Answer
1






active

oldest

votes









active

oldest

votes






active

oldest

votes








up vote
0
down vote



accepted










It follows easily from the version of Harnak's inequality here that if $K$ is a compact subset of $Bbb D=D(0,1)$ there exist $c_K$ and $C_K$ such that
$$c_Ku(0)le u(z)le C_Ku(0)quad(zin K)$$for every positive harmonic function $u$ in $Bbb D$.



So if $Re f_n(0)toinfty$ then $|f_n(z)|toinfty$ uniformly on compact sets. Otoh if $Re f_n(0)nottoinfty$ then restricting to a subsequence we can assume that $Re f_n(0)$ is bounded; now the inequality above shows that $e^f_n$ is uniformly bounded on compact sets...



Heh, a slick way to get around unpleasantness with the logarithm for the punchline: So again passing to a subequence we can assume that $e^f_nto g$ uniformly on compact sets. Since $left|e^f_nright|ge1$ it follows that $g$ has no zero, so $g=e^f$. How to get from there to $f_nto f$: Since normal convergence implies normal convergence of the derivatives we have $f_n'e^f_nto f'e^f$, and now since $left|e^f_nright|ge1$ it follows that $f_n'to f'$ uniformly on compact sets. So if we choose our logarithms so that $f_n(0)to f(0)$ it follows that $f_nto f$ uniformly on compact sets.






share|cite|improve this answer























  • Why not just go directly to $e^-f_n $ is uniformly bounded?
    – zhw.
    Jul 16 at 23:46










  • @zhw. This is what they call a dense argument...
    – David C. Ullrich
    Jul 16 at 23:48










  • @zhw. Actually I have a better reason than I did when I wrote the first version. Saw a cute way to handle the technicalities at the end; for that to work with the $e^-f_n$ version I need to know that $|e^-f_n|ge c>0$ on compact sets.
    – David C. Ullrich
    Jul 16 at 23:58











  • @Rachel.L Seriously? $g$ is holomorphic in a disk and has no zero, so it has a holomorphic logarithm...
    – David C. Ullrich
    Jul 24 at 20:57














up vote
0
down vote



accepted










It follows easily from the version of Harnak's inequality here that if $K$ is a compact subset of $Bbb D=D(0,1)$ there exist $c_K$ and $C_K$ such that
$$c_Ku(0)le u(z)le C_Ku(0)quad(zin K)$$for every positive harmonic function $u$ in $Bbb D$.



So if $Re f_n(0)toinfty$ then $|f_n(z)|toinfty$ uniformly on compact sets. Otoh if $Re f_n(0)nottoinfty$ then restricting to a subsequence we can assume that $Re f_n(0)$ is bounded; now the inequality above shows that $e^f_n$ is uniformly bounded on compact sets...



Heh, a slick way to get around unpleasantness with the logarithm for the punchline: So again passing to a subequence we can assume that $e^f_nto g$ uniformly on compact sets. Since $left|e^f_nright|ge1$ it follows that $g$ has no zero, so $g=e^f$. How to get from there to $f_nto f$: Since normal convergence implies normal convergence of the derivatives we have $f_n'e^f_nto f'e^f$, and now since $left|e^f_nright|ge1$ it follows that $f_n'to f'$ uniformly on compact sets. So if we choose our logarithms so that $f_n(0)to f(0)$ it follows that $f_nto f$ uniformly on compact sets.






share|cite|improve this answer























  • Why not just go directly to $e^-f_n $ is uniformly bounded?
    – zhw.
    Jul 16 at 23:46










  • @zhw. This is what they call a dense argument...
    – David C. Ullrich
    Jul 16 at 23:48










  • @zhw. Actually I have a better reason than I did when I wrote the first version. Saw a cute way to handle the technicalities at the end; for that to work with the $e^-f_n$ version I need to know that $|e^-f_n|ge c>0$ on compact sets.
    – David C. Ullrich
    Jul 16 at 23:58











  • @Rachel.L Seriously? $g$ is holomorphic in a disk and has no zero, so it has a holomorphic logarithm...
    – David C. Ullrich
    Jul 24 at 20:57












up vote
0
down vote



accepted







up vote
0
down vote



accepted






It follows easily from the version of Harnak's inequality here that if $K$ is a compact subset of $Bbb D=D(0,1)$ there exist $c_K$ and $C_K$ such that
$$c_Ku(0)le u(z)le C_Ku(0)quad(zin K)$$for every positive harmonic function $u$ in $Bbb D$.



So if $Re f_n(0)toinfty$ then $|f_n(z)|toinfty$ uniformly on compact sets. Otoh if $Re f_n(0)nottoinfty$ then restricting to a subsequence we can assume that $Re f_n(0)$ is bounded; now the inequality above shows that $e^f_n$ is uniformly bounded on compact sets...



Heh, a slick way to get around unpleasantness with the logarithm for the punchline: So again passing to a subequence we can assume that $e^f_nto g$ uniformly on compact sets. Since $left|e^f_nright|ge1$ it follows that $g$ has no zero, so $g=e^f$. How to get from there to $f_nto f$: Since normal convergence implies normal convergence of the derivatives we have $f_n'e^f_nto f'e^f$, and now since $left|e^f_nright|ge1$ it follows that $f_n'to f'$ uniformly on compact sets. So if we choose our logarithms so that $f_n(0)to f(0)$ it follows that $f_nto f$ uniformly on compact sets.






share|cite|improve this answer















It follows easily from the version of Harnak's inequality here that if $K$ is a compact subset of $Bbb D=D(0,1)$ there exist $c_K$ and $C_K$ such that
$$c_Ku(0)le u(z)le C_Ku(0)quad(zin K)$$for every positive harmonic function $u$ in $Bbb D$.



So if $Re f_n(0)toinfty$ then $|f_n(z)|toinfty$ uniformly on compact sets. Otoh if $Re f_n(0)nottoinfty$ then restricting to a subsequence we can assume that $Re f_n(0)$ is bounded; now the inequality above shows that $e^f_n$ is uniformly bounded on compact sets...



Heh, a slick way to get around unpleasantness with the logarithm for the punchline: So again passing to a subequence we can assume that $e^f_nto g$ uniformly on compact sets. Since $left|e^f_nright|ge1$ it follows that $g$ has no zero, so $g=e^f$. How to get from there to $f_nto f$: Since normal convergence implies normal convergence of the derivatives we have $f_n'e^f_nto f'e^f$, and now since $left|e^f_nright|ge1$ it follows that $f_n'to f'$ uniformly on compact sets. So if we choose our logarithms so that $f_n(0)to f(0)$ it follows that $f_nto f$ uniformly on compact sets.







share|cite|improve this answer















share|cite|improve this answer



share|cite|improve this answer








edited Jul 16 at 23:55


























answered Jul 16 at 23:34









David C. Ullrich

54.3k33583




54.3k33583











  • Why not just go directly to $e^-f_n $ is uniformly bounded?
    – zhw.
    Jul 16 at 23:46










  • @zhw. This is what they call a dense argument...
    – David C. Ullrich
    Jul 16 at 23:48










  • @zhw. Actually I have a better reason than I did when I wrote the first version. Saw a cute way to handle the technicalities at the end; for that to work with the $e^-f_n$ version I need to know that $|e^-f_n|ge c>0$ on compact sets.
    – David C. Ullrich
    Jul 16 at 23:58











  • @Rachel.L Seriously? $g$ is holomorphic in a disk and has no zero, so it has a holomorphic logarithm...
    – David C. Ullrich
    Jul 24 at 20:57
















  • Why not just go directly to $e^-f_n $ is uniformly bounded?
    – zhw.
    Jul 16 at 23:46










  • @zhw. This is what they call a dense argument...
    – David C. Ullrich
    Jul 16 at 23:48










  • @zhw. Actually I have a better reason than I did when I wrote the first version. Saw a cute way to handle the technicalities at the end; for that to work with the $e^-f_n$ version I need to know that $|e^-f_n|ge c>0$ on compact sets.
    – David C. Ullrich
    Jul 16 at 23:58











  • @Rachel.L Seriously? $g$ is holomorphic in a disk and has no zero, so it has a holomorphic logarithm...
    – David C. Ullrich
    Jul 24 at 20:57















Why not just go directly to $e^-f_n $ is uniformly bounded?
– zhw.
Jul 16 at 23:46




Why not just go directly to $e^-f_n $ is uniformly bounded?
– zhw.
Jul 16 at 23:46












@zhw. This is what they call a dense argument...
– David C. Ullrich
Jul 16 at 23:48




@zhw. This is what they call a dense argument...
– David C. Ullrich
Jul 16 at 23:48












@zhw. Actually I have a better reason than I did when I wrote the first version. Saw a cute way to handle the technicalities at the end; for that to work with the $e^-f_n$ version I need to know that $|e^-f_n|ge c>0$ on compact sets.
– David C. Ullrich
Jul 16 at 23:58





@zhw. Actually I have a better reason than I did when I wrote the first version. Saw a cute way to handle the technicalities at the end; for that to work with the $e^-f_n$ version I need to know that $|e^-f_n|ge c>0$ on compact sets.
– David C. Ullrich
Jul 16 at 23:58













@Rachel.L Seriously? $g$ is holomorphic in a disk and has no zero, so it has a holomorphic logarithm...
– David C. Ullrich
Jul 24 at 20:57




@Rachel.L Seriously? $g$ is holomorphic in a disk and has no zero, so it has a holomorphic logarithm...
– David C. Ullrich
Jul 24 at 20:57












 

draft saved


draft discarded


























 


draft saved


draft discarded














StackExchange.ready(
function ()
StackExchange.openid.initPostLogin('.new-post-login', 'https%3a%2f%2fmath.stackexchange.com%2fquestions%2f2853554%2fa-sequence-of-holomorphic-functions-in-d0-1%23new-answer', 'question_page');

);

Post as a guest













































































Comments

Popular posts from this blog

Color the edges and diagonals of a regular polygon

Relationship between determinant of matrix and determinant of adjoint?

What is the equation of a 3D cone with generalised tilt?